Diễn Đàn MathScopeDiễn Đàn MathScope
  Diễn Đàn MathScope
Ghi Danh Hỏi/Ðáp Thành Viên Social Groups Lịch Ðánh Dấu Ðã Ðọc

Go Back   Diễn Đàn MathScope > Sơ Cấp > Việt Nam và IMO > 2014

News & Announcements

Ngoài một số quy định đã được nêu trong phần Quy định của Ghi Danh , mọi người tranh thủ bỏ ra 5 phút để đọc thêm một số Quy định sau để khỏi bị treo nick ở MathScope nhé !

* Nội quy MathScope.Org

* Một số quy định chung !

* Quy định về việc viết bài trong diễn đàn MathScope

* Nếu bạn muốn gia nhập đội ngũ BQT thì vui lòng tham gia tại đây

* Những câu hỏi thường gặp

* Về việc viết bài trong Box Đại học và Sau đại học


Trả lời Gởi Ðề Tài Mới
 
Ðiều Chỉnh Xếp Bài
Old 24-03-2014, 02:36 AM   #1
huynhcongbang
Administrator

 
huynhcongbang's Avatar
 
Tham gia ngày: Feb 2009
Đến từ: Ho Chi Minh City
Bài gởi: 2,413
Thanks: 2,165
Thanked 4,188 Times in 1,381 Posts
Gửi tin nhắn qua Yahoo chát tới huynhcongbang
Việt Nam Team Selection Test 2014 - Đề thi, lời giải và danh sách đội tuyển


Sáng nay, ngày 24/03/2014, 5 đội tuyển Toán, Vật Lý, Hóa Học, Sinh Học, Tin Học sẽ tập trung về ĐH Sư phạm Hà Nội để tham dự lễ khai mạc cho kỳ thi chọn đội tuyển dự thi Olympic Quốc tế. Trong năm 2014, thời gian tổ chức của các kỳ thi quốc tế đối với 5 môn KHTN này như sau:

- Từ 06-13/07, Olympic Sinh học quốc tế, IBO lần thứ 25 ở Bali, Indonesia.
- Từ 13-20/07, Olympic Tin học quốc tế, IOI lần thứ 26, Taipei, Đài Loan.
- Từ 20-29/07, Olympic Hóa học quốc tế, IChO lần thứ 46, Hà Nội, Việt Nam.
- Từ 13-21/07, Olympic Vật lý quốc tế, IPhO lần thứ 45, tại Astana, Kazakhstan.
- Từ 03-13/07, Olympic Toán quốc tế, IMO lần thứ 55, tại Cape Town, South Africa.

Như thế thì trong 5 môn, IMO được tổ chức sớm nhất và sớm hơn hẳn các năm đến 2-3 tuần. Dưới đây là trang web chính thức của kỳ thi:


Kỳ thi chọn đội tuyển chính thức sẽ diễn ra vào ngày sau đó là 25/03. Cho đến thời điểm này, có lẽ các bạn học sinh dự thi Toán của các đội tuyển đã tập trung đầy đủ tại Hà Nội. Trong tổng số 48 thí sinh tham gia để chọn ra 6 học sinh cho đội tuyển chính thức thi IMO tại Nam Phi, có 46 học sinh giải Nhì có điểm từ 24.75đ trở lên trong kỳ thi HSGQG vừa qua cùng 2 học sinh đã vào đội dự tuyển năm trước. Đặc biệt, số lượng thí sinh nữ tham gia trong năm nay tăng hơn so với năm ngoái (từ 2/45 lên 10/48), trong đó có đến 3 bạn có giải Nhất.

Dưới đây là danh sách đầy đủ:



Nếu không có gì thay đổi thì các bạn thí sinh vẫn sẽ thi trong vòng 2 ngày, mỗi ngày giải 3 bài Toán trong vòng 270 phút với 4 phân môn: Đại số, Hình học, Số học và Tổ hợp. Vẫn như các năm, chúng ta đang trông đợi một đề thi mang tính chuyên môn cao với nhiều bài Toán thử thách, mới mẻ cùng nhiều bất ngờ thú vị.

Topic này để mọi người gửi đề bài, giải các bài toán trong đề thi và thảo luận, đóng góp các thông tin về kì thi này. Đến trưa mỗi ngày thi thì chắc chắn sẽ có đề bài đầy đủ. Rất mong nhận được sự quan tâm, trao đổi thảo luận của các bạn yêu Toán để có các lời giải phong phú, bình luận ý nghĩa. Chúc các bạn thí sinh có tâm lí vững vàng, phong độ đỉnh cao để làm bài hết khả năng của mình và giành giải cao nhất có thể!
[RIGHT][I][B]Nguồn: MathScope.ORG[/B][/I][/RIGHT]
 
__________________
Sự im lặng của bầy mèo
huynhcongbang is offline   Trả Lời Với Trích Dẫn
The Following 23 Users Say Thank You to huynhcongbang For This Useful Post:
blackholes. (24-03-2014), dangvip123tb (24-03-2014), hoangqnvip (24-03-2014), khi gia (24-03-2014), kimlinh (27-03-2014), lenguyenA1 (25-03-2014), lion (24-03-2014), Littlemonster (24-03-2014), madman (24-03-2014), mathandyou (24-03-2014), MathForLife (27-03-2014), nam8298 (25-03-2014), namdung (24-03-2014), Nguyen Van Linh (24-03-2014), nguyentatthu (24-03-2014), phucbentre (24-03-2014), ptnkmt11 (24-03-2014), thaygiaocht (24-03-2014), thiendieu96 (24-03-2014), thuongdinh (24-03-2014), vulalach (26-03-2014), whatever2507 (24-03-2014), WhjteShadow (24-03-2014)
Old 24-03-2014, 09:36 AM   #2
ptnkmt11
+Thành Viên+
 
Tham gia ngày: Jun 2012
Bài gởi: 75
Thanks: 48
Thanked 31 Times in 24 Posts
Chúc các thí sinh đạt phong độ cao nhất!!!
[RIGHT][I][B]Nguồn: MathScope.ORG[/B][/I][/RIGHT]
 
ptnkmt11 is offline   Trả Lời Với Trích Dẫn
Old 24-03-2014, 12:16 PM   #3
hung_020297
+Thành Viên+
 
Tham gia ngày: Mar 2012
Đến từ: thanh hoa roi
Bài gởi: 45
Thanks: 15
Thanked 1 Time in 1 Post
Chúc các bạn nữ thi tốt
[RIGHT][I][B]Nguồn: MathScope.ORG[/B][/I][/RIGHT]
 
hung_020297 is offline   Trả Lời Với Trích Dẫn
Old 24-03-2014, 03:49 PM   #4
mathandyou
Moderator
 
Tham gia ngày: Dec 2012
Đến từ: HCMUS
Bài gởi: 557
Thanks: 259
Thanked 402 Times in 216 Posts
Dự đoán đề đi các bạn.
Mình dự là:
[RIGHT][I][B]Nguồn: MathScope.ORG[/B][/I][/RIGHT]
 
__________________
Xét cho cùng, phần thưởng cao quý nhất mà công việc mang lại không phải là thứ bạn nhận được, mà nó vẽ nên chân dung con người bạn ra sao.

[Only registered and activated users can see links. ]
mathandyou is offline   Trả Lời Với Trích Dẫn
Old 25-03-2014, 02:08 PM   #5
huynhcongbang
Administrator

 
huynhcongbang's Avatar
 
Tham gia ngày: Feb 2009
Đến từ: Ho Chi Minh City
Bài gởi: 2,413
Thanks: 2,165
Thanked 4,188 Times in 1,381 Posts
Gửi tin nhắn qua Yahoo chát tới huynhcongbang
Gửi mọi người đề thi ngày thứ nhất, nguồn: blackhole.

Ngày thi thứ nhất (25/03/2014)

Bài 1.
Tìm tất cả các hàm số $f:\mathbb{Z}\to \mathbb{Z}$ thỏa mãn điều kiện $$f\left( 2m+f(m)+f(m)f(n) \right)=nf(m)+m$$ với mọi $m,n$ là các số nguyên.

Bài 2.
Trong mặt phẳng tọa độ $Oxy$, xét các điểm nguyên có tọa độ thuộc $$T=\left\{ (x;y):\left| x \right|,\left| y \right|\le 20,(x;y)\ne (0;0) \right\}.$$ Tô màu các điểm thuộc $T$ sao cho với mọi điểm có tọa độ $(x,y)\in T$ thì có đúng một trong hai điểm $(x;y)$ và $(-x;-y)$ được tô màu. Với mỗi cách tô như thế, gọi $N$ là số các bộ $({{x}_{1}};{{y}_{1}}),({{x}_{2}};{{y}_{2}})$ mà cả hai điểm này cùng được tô màu và ${{x}_{1}}\equiv 2{{x}_{2}},{{y}_{1}}\equiv 2{{y}_{2}}(\bmod 41)$. Tìm tất cả các giá trị có thể có của $N.$

Bài 3.
Cho tam giác $ABC$ có $A<B<C$ và nội tiếp trong đường tròn $(O).$ Trên cung nhỏ $BC$ của $(O)$ và không chứa điểm $A$, lấy điểm $D$ tùy ý. Giả sử $CD$ cắt $AB$ ở $E$ và $BD$ cắt $AC$ ở $F$. Gọi ${{O}_{1}}$ là tâm đường tròn nằm trong tam giác $EBD$, tiếp xúc với $EB,ED$ và tiếp xúc với đường tròn $(O).$ Gọi ${{O}_{2}}$ là tâm đường tròn nằm trong tam giác $FCD$, tiếp xúc với $FC,FD$ và tiếp xúc với đường tròn $(O).$
a. Gọi $M$ là tiếp điểm của $({{O}_{1}})$ với $BE$ và $N$ là tiếp điểm của ${{O}_{2}}$ với $CF$. Chứng minh rằng đường tròn đường kính $MN$ luôn đi qua một điểm cố định.
b. Đường thẳng qua $M$ và song song với $CE$ cắt $AC$ ở $P$, đường thẳng qua $N$ và song song với $BF$cắt $AB$ ở $Q$. Chứng minh rằng đường tròn ngoại tiếp tam giác $(AMP),(ANQ)$ cùng tiếp xúc với một đường tròn cố định.
[RIGHT][I][B]Nguồn: MathScope.ORG[/B][/I][/RIGHT]
 
File Kèm Theo
Kiểu File : pdf De TST 2014 Ngay 1.pdf (143.2 KB, 364 lần tải)
__________________
Sự im lặng của bầy mèo

thay đổi nội dung bởi: huynhcongbang, 25-03-2014 lúc 02:35 PM
huynhcongbang is offline   Trả Lời Với Trích Dẫn
The Following 15 Users Say Thank You to huynhcongbang For This Useful Post:
blackholes. (25-03-2014), dangvip123tb (25-03-2014), einstein1996 (25-03-2014), lenguyenA1 (25-03-2014), mathandyou (25-03-2014), nam8298 (25-03-2014), nghiepdu-socap (25-03-2014), nqt (25-03-2014), ptnkmt11 (25-03-2014), q785412369 (25-03-2014), quykhtn (25-03-2014), thaygiaocht (25-03-2014), thiendieu96 (25-03-2014), vanchay (25-03-2014), vinhhop.qt (25-03-2014)
Old 25-03-2014, 03:57 PM   #6
CanVQ
+Thành Viên+
 
Tham gia ngày: Jan 2014
Bài gởi: 7
Thanks: 4
Thanked 18 Times in 7 Posts
Bài hàm là bài dễ nhất của đề thi hôm nay. Các bạn có thể tham khảo lời giải ở đây: [Only registered and activated users can see links. ]
[RIGHT][I][B]Nguồn: MathScope.ORG[/B][/I][/RIGHT]
 
CanVQ is offline   Trả Lời Với Trích Dẫn
The Following 4 Users Say Thank You to CanVQ For This Useful Post:
dangvip123tb (25-03-2014), huynhcongbang (25-03-2014), nam8298 (25-03-2014), vanchay (25-03-2014)
Old 25-03-2014, 03:57 PM   #7
chemthan
Administrator

 
chemthan's Avatar
 
Tham gia ngày: Mar 2009
Bài gởi: 349
Thanks: 0
Thanked 308 Times in 161 Posts
Trích:
Nguyên văn bởi huynhcongbang View Post
Bài 2.
Trong mặt phẳng tọa độ $Oxy$, xét các điểm nguyên có tọa độ thuộc $$T=\left\{ (x;y):\left| x \right|,\left| y \right|\le 20,(x;y)\ne (0;0) \right\}.$$ Tô màu các điểm thuộc $T$ sao cho với mọi điểm có tọa độ $(x,y)\in T$ thì có đúng một trong hai điểm $(x;y)$ và $(-x;-y)$ được tô màu. Với mỗi cách tô như thế, gọi $N$ là số các bộ $({{x}_{1}};{{y}_{1}}),({{x}_{2}};{{y}_{2}})$ mà cả hai điểm này cùng được tô màu và ${{x}_{1}}\equiv 2{{x}_{2}},{{y}_{1}}\equiv 2{{y}_{2}}(\bmod 41)$. Tìm tất cả các giá trị có thể có của $N.$
Tập $T $ có thể phân hoạch thành 84 tập con $T_1, T_2,..., T_{42} $. Trong đó mỗi tập con gồm 20 phần tử $\{(x_1, y_1), (x_2, y_2), ..., (x_{20}, y_{20})\} $ thỏa mãn $x_{i+1}\equiv 2x_i $ (mod 41), $y_{i+1}\equiv 2y_i $ (mod 41).
Vì $2^{10} \equiv -1 $ (mod 41) nên trong 2 điểm $(x_i, y_i), (x_{i+10}, y_{i+10}) $ có đúng 1 điểm được tô màu.
Do đó số các số bộ $(x_i, y_i), (x_{i+1}, y_{i+1}) $ cùng được tô màu có thể là $1, 3, 5, 7, 9 $.
Từ đó $N\in \{84, 86, ..., 84 * 9\} $.
[RIGHT][I][B]Nguồn: MathScope.ORG[/B][/I][/RIGHT]
 

thay đổi nội dung bởi: chemthan, 25-03-2014 lúc 04:14 PM
chemthan is offline   Trả Lời Với Trích Dẫn
The Following 9 Users Say Thank You to chemthan For This Useful Post:
dangvip123tb (25-03-2014), Fool's theorem (25-03-2014), huynhcongbang (25-03-2014), luugiangnam (19-07-2014), nam8298 (25-03-2014), ntuan5 (10-04-2014), quocbaoct10 (26-03-2014), thiendieu96 (25-03-2014), whatever2507 (25-03-2014)
Old 25-03-2014, 04:11 PM   #8
Nguyen Van Linh
Moderator
 
Tham gia ngày: Aug 2009
Đến từ: Hà Nội
Bài gởi: 277
Thanks: 69
Thanked 323 Times in 145 Posts
Bài hình năm nay khá hay.

a) Gọi $J$ là tâm đường tròn bàng tiếp góc $A$ của tam giác $ABC$. Gọi $\omega_1, \omega_2$ lần lượt là đường tròn tiếp xúc với $EB, ED$; $FC,FD$ và tiếp xúc với $(O), L,K$ là tiếp điểm của $\omega_1$ với $ED, \omega_2$ với $FD$. Theo định lý Sawayama-Thebault thì $M,L,J$ thẳng hàng và $N,K,J$ thẳng hàng. Bằng phép cộng góc đơn giản suy ra phân giác các góc $BED$ và $CFD$ vuông góc với nhau, suy ra $\angle MJN=90^\circ$, tức là đường tròn đường kính $MN$ luôn đi qua $J$ cố định.
b) $MP\parallel EC$ nên $\angle EML=\angle MLE=\angle LMP$, hay $MJ$ là phân giác $\angle EMP$, tức là $J$ cũng là tâm bàng tiếp tam giác $AMP$, suy ra $(AMP)$ luôn tiếp xúc với đường tròn Mixtilinear bàng tiếp góc $A$ của tam giác $ABC.$ Tương tự với $(ANQ)$.

Bài này nhất định còn nhiều thứ thú vị bên trong để khai thác
[RIGHT][I][B]Nguồn: MathScope.ORG[/B][/I][/RIGHT]
 
File Kèm Theo
Kiểu File : pdf TST3.2014.pdf (27.3 KB, 316 lần tải)
Nguyen Van Linh is offline   Trả Lời Với Trích Dẫn
The Following 7 Users Say Thank You to Nguyen Van Linh For This Useful Post:
dangvip123tb (25-03-2014), huynhcongbang (25-03-2014), Manhnguyen (25-03-2014), nam8298 (25-03-2014), thaygiaocht (25-03-2014), thiendieu96 (25-03-2014), whatever2507 (25-03-2014)
Old 25-03-2014, 06:20 PM   #9
tikita
Administrator

 
Tham gia ngày: Jun 2012
Bài gởi: 157
Thanks: 2
Thanked 84 Times in 53 Posts
Trích:
Nguyên văn bởi huynhcongbang View Post
Gửi mọi người đề thi ngày thứ nhất, nguồn: blackhole.

Ngày thi thứ nhất (25/03/2014)

Bài 2.
Trong mặt phẳng tọa độ $Oxy$, xét các điểm nguyên có tọa độ thuộc $$T=\left\{ (x;y):\left| x \right|,\left| y \right|\le 20,(x;y)\ne (0;0) \right\}.$$ Tô màu các điểm thuộc $T$ sao cho với mọi điểm có tọa độ $(x,y)\in T$ thì có đúng một trong hai điểm $(x;y)$ và $(-x;-y)$ được tô màu. Với mỗi cách tô như thế, gọi $N$ là số các bộ $({{x}_{1}};{{y}_{1}}),({{x}_{2}};{{y}_{2}})$ mà cả hai điểm này cùng được tô màu và ${{x}_{1}}\equiv 2{{x}_{2}},{{y}_{1}}\equiv 2{{y}_{2}}(\bmod 41)$. Tìm tất cả các giá trị có thể có của $N.$
Chú ý rằng với mỗi $x_2\in [-20,20]$ luôn tồn tại duy nhất $x_1$ sao cho $x_1\equiv 2x_2 (\bmod 41)$. Từ đây suy ra với mỗi điểm $(x_2,y_2)\in T$ luôn tồn tại duy nhất một điểm $(x_1,y_1)$ sao cho ${{x}_{1}}\equiv 2{{x}_{2}},{{y}_{1}}\equiv 2{{y}_{2}}(\bmod 41)$. Điều này có nghĩa là ta có thể phân hoạch các điểm trong $T$ thành bộ gồm bốn điểm $\{(x_1,y_1),(x_2,y_2),(-x_1,-y_1),(-x_2,-y_2)\}$ với các bộ không có phần tử chụng Rỏ ràng với cách tô màu theo đề bài thì mỗi bộ có tối đa một cặp cùng màu thỏa yêu cầu bài toạn Vậy số $N\in [0,420]$.
[RIGHT][I][B]Nguồn: MathScope.ORG[/B][/I][/RIGHT]
 
tikita is offline   Trả Lời Với Trích Dẫn
The Following User Says Thank You to tikita For This Useful Post:
bachhammer (29-03-2014)
Old 25-03-2014, 06:48 PM   #10
dangvip123tb
+Thành Viên+
 
Tham gia ngày: Jul 2012
Đến từ: hanoi
Bài gởi: 17
Thanks: 165
Thanked 5 Times in 5 Posts
Trích:
Nguyên văn bởi tikita View Post
Chú ý rằng với mỗi $x_2\in [-20,20]$ luôn tồn tại duy nhất $x_1$ sao cho $x_1\equiv 2x_2 (\bmod 41)$. Từ đây suy ra với mỗi điểm $(x_2,y_2)\in T$ luôn tồn tại duy nhất một điểm $(x_1,y_1)$ sao cho ${{x}_{1}}\equiv 2{{x}_{2}},{{y}_{1}}\equiv 2{{y}_{2}}(\bmod 41)$. Điều này có nghĩa là ta có thể phân hoạch các điểm trong $T$ thành bộ gồm bốn điểm $\{(x_1,y_1),(x_2,y_2),(-x_1,-y_1),(-x_2,-y_2)\}$ với các bộ không có phần tử chụng Rỏ ràng với cách tô màu theo đề bài thì mỗi bộ có tối đa một cặp cùng màu thỏa yêu cầu bài toạn Vậy số $N\in [0,420]$.
bạn làm nhầm rồi. hôm nay mình cũng làm như vậy.
nhưng cách làm của anh chemthan mới đúng.
[RIGHT][I][B]Nguồn: MathScope.ORG[/B][/I][/RIGHT]
 
dangvip123tb is offline   Trả Lời Với Trích Dẫn
Old 25-03-2014, 07:23 PM   #11
Nguyen Van Linh
Moderator
 
Tham gia ngày: Aug 2009
Đến từ: Hà Nội
Bài gởi: 277
Thanks: 69
Thanked 323 Times in 145 Posts
Trích:
Nguyên văn bởi BlackSelena View Post
Hình như cái này ko giống Sawayama cho lắm...
Bạn nên nhìn rộng ra, Sawayama nó áp dụng cho nhiều trường hợp, cả trong cả ngoài, không phải chỉ mỗi trong.
[RIGHT][I][B]Nguồn: MathScope.ORG[/B][/I][/RIGHT]
 
Nguyen Van Linh is offline   Trả Lời Với Trích Dẫn
Old 26-03-2014, 04:22 AM   #12
huynhcongbang
Administrator

 
huynhcongbang's Avatar
 
Tham gia ngày: Feb 2009
Đến từ: Ho Chi Minh City
Bài gởi: 2,413
Thanks: 2,165
Thanked 4,188 Times in 1,381 Posts
Gửi tin nhắn qua Yahoo chát tới huynhcongbang
Mình thử phân tích thêm lời giải cho bài 2 như sau:

Trong bài toán đã cho, ta thấy hoành độ và tung độ hoàn toàn độc lập với nhau nên để dễ hình dung, có thể xét riêng thành phần hoành độ.

Trước hết, ta viết các số từ $-20$ đến $20$ (bỏ số $0$ ra) thành các dãy mà liền sau số $x$ là số $2x$ theo $\mod 41$. Khi đó, ta được 2 dãy như bên dưới: $$A: 1,2,4,8,16,-9,-18,5,10,20,-1,-2,-4,-8,-16,9,18,-5,-10,-20,1$$ $$B: 3,6,12,-17,7,14,-13,15,-11,19,-3,-6,-12,17,-7,-14,13,-15,11,-19,3$$ Tô màu các số của dãy này sao cho trong 2 số đối nhau thì có đúng 1 số được tô màu. Ta quan tâm đến số lượng các cặp số liên tiếp cùng được tô màu trong dãy.

Do có sự lặp lại ở cả 2 dãy nên để dễ hình dung, ta chuyển thành vòng tròn và phát biểu lại bài toán như sau:

Cho 20 điểm chia đều một đường tròn sao cho trong 2 điểm đối xứng qua tâm thì có đúng 1 đỉnh được tô màu. Tính số các cặp đỉnh liên tiếp được tô màu có thể có.

Xét các trường hợp số đỉnh chẵn nhỏ hơn:
- Đa giác 4 đỉnh: 1 cặp.
- Đa giác 6 đỉnh: 0-2 cặp.
- Đa giác 8 đỉnh: 1-3 cặp.
- Đa giác 10 đỉnh: 0-2-4 cặp.
...
Như thế, dự đoán tổng quát là:
Với $n$ là số chẵn, gọi $S_{n}$ là tập hợp số các cặp kề nhau cùng được tô màu có thể có của đa giác có $n$ đỉnh thì:
$S_{4n} = \{ 2k+1 | 0 \le k \le n-1 \} $ và $S_{4n-2} = \{ 2k | 0 \le k \le n-1 \} $.

Điều này có thể chứng minh bằng quy nạp như sau:
- Với $n=2$ thì nhận xét đúng.
- Giả sử đúng đến $n.$
Xét đa giác có $4n+2$ đỉnh. Đa giác này có thể tạo thành bằng cách thêm đỉnh $A$ vào giữa hai đỉnh thứ 2n, 2n+1 và thêm đỉnh $B$ vào giữa hai đỉnh thứ 4n, 1. Ta xét các trường hợp:
- Nếu đỉnh $2n$ và $2n+1$ đều được tô và $A$ không được tô thì tương ứng: đỉnh $4n, 1$ không được tô và $B$ được tô. Số cặp kề nhau cùng được tô giảm đi 1.
- Nếu đỉnh $2n$ và $2n+1$ đều được tô và $A$ cũng được tô thì tương ứng: đỉnh $4n, 1$ không được tô và $B$ không được tô. Số cặp kề nhau tăng lên 1.
- Nếu trong hai đỉnh $2n$ và $2n+1,$ có 1 đỉnh được tô và $A$ cũng được tô thì tương ứng: trong hai đỉnh 4n, 1 có 1 đỉnh được tô và $B$ không được tô. Số cặp kề nhau tăng lên 1.
- Nếu trong hai đỉnh $2n$ và $2n+1,$ có 1 đỉnh được tô và $A$ không được tô thì tương ứng: trong hai đỉnh 4n, 1 có 1 đỉnh được tô và $B$ được tô. Số cặp kề nhau tăng lên 1.

Do đó, $S_{4n+2} = \{ x \pm 1 | x \in S_{4n} \} $ hay $S_{4n+2}=\{ 2k | 0 \le k \le n \}$.
Tương tự, ta cũng có $S_{4n+4} = \{ x \pm 1 | x \in S_{4n+2} \} $. Tất nhiên không xảy ra trường hợp $S_{4n+4}$ có chứa số $-1$ vì để có trường hợp 0 cặp số ở $4n+2$, các đỉnh phải được tô xen kẽ và trường hợp giảm đi số bộ không xảy ra, suy ra $S_{4n+4}=\{ 2k+1 | 0 \le k \le n \}.$
Nhận xét được chứng minh.

Từ đó suy ra $S_{20}=\{ 1, 3, 5, 7, 9 \}$.
Do đó, kết quả cho bài toán phụ sẽ là $2S_{20}$ với định nghĩa $2S=S+S=\{a+b|a,b \in S \}$.

Quay trở lại bài toán ban đầu, để chuyển từ 1 thành phần $x$ ở trên thành 2 thành phần $(x,y)$, ta có thể hình dung như sau:

Ứng với mỗi vòng tròn chứa các số thuộc dãy $A$, ta lấy một vòng tròn mới cũng gồm các số thuộc dãy $A$ đặt lên đó sao cho mỗi số thuộc đường tròn cũ khớp với một số thuộc đường tròn mới. Viết các cặp số khớp nhau thành một dãy, dãy đó chính là dãy các tọa độ điểm mà liền sau của $(x_1, y_1)$ là $(2x_1, 2y_1)$ theo $\mod 41$.

Dễ thấy có tất cả 20 cách ghép như thế (cố định vòng tròn cũ và xoay vòng tròn mới). Tương tự với việc ghép các dãy $A-B, B-A$ và $B-B$ nên có tổng cộng là 80 cách ghép tạo thành 80 dãy.

Tuy nhiên, ta cũng xét thêm 4 dãy đặc biệt, tương ứng với các điểm nằm trên trục tung và trục hoành.
Cụ thể là xét thêm dãy C gồm 20 số 0 và xét 4 cách ghép: $A-C, C-A, B-C, C-B.$

Do đó, tổng cộng có 84 dãy các tọa độ.

Theo chứng minh ở trên thì ở mỗi dãy, số các cặp có thể có là $S_{20}$ vậy nên đáp số của bài toán là $84S_{20}$, cũng chính là các số chẵn từ $84$ đến $9 \cdot 84$.

Bài toán đến đây là kết thúc.

Ta thử đặt vấn đề trong trường hợp thay 20 và 41 bởi các số khác.
Chẳng hạn nếu thay 41 bởi 11 thì thay vì có 84 dãy mà mỗi dãy độ dài 20, ta sẽ có đến 168 dãy mà mỗi dãy có độ dài là 10. Đáp số sẽ là $168S_{10}$.
Các số 41, 11 quyết định độ dài chu kỳ của các dãy ở trên. Tất nhiên, có những trường hợp dãy không lặp lại thì phải tính toán theo cách khác nhưng nói chung vẫn khả thi.

Theo nhận xét của mình thì bài toán này có phong cách của bài 4 VMO 2012, bài 3 VMO 2013, đều là các bài khá mới mẻ, là dạng tổ hợp đếm có kết hợp số học, tuy phát biểu hơi có phần gượng ép trong việc đặt vào trục tọa độ nhưng lại giúp học sinh dễ tưởng tượng hơn.
[RIGHT][I][B]Nguồn: MathScope.ORG[/B][/I][/RIGHT]
 
__________________
Sự im lặng của bầy mèo

thay đổi nội dung bởi: huynhcongbang, 26-03-2014 lúc 04:37 AM
huynhcongbang is offline   Trả Lời Với Trích Dẫn
The Following 2 Users Say Thank You to huynhcongbang For This Useful Post:
thiendieu96 (26-03-2014), tuandaisu (26-03-2014)
Old 26-03-2014, 03:17 PM   #13
huynhcongbang
Administrator

 
huynhcongbang's Avatar
 
Tham gia ngày: Feb 2009
Đến từ: Ho Chi Minh City
Bài gởi: 2,413
Thanks: 2,165
Thanked 4,188 Times in 1,381 Posts
Gửi tin nhắn qua Yahoo chát tới huynhcongbang
Đề thi ngày 2.

Bài 4.
a. Cho tam giác $ABC$ có đường cao $AD$ và $P$ là một điểm di động trên $AD$. Các đường thẳng $PB$ và $AC$ cắt nhau ở $E$, các đường thẳng $PC$ và $AB$ cắt nhau ở $F.$ Giả sử tứ giác $AEDF$ nội tiếp. Chứng minh rằng $$\frac{PA}{PD}=(\tan B+\tan C)\cot \frac{A}{2}.$$ b. Cho tam giác $ABC$ có trực tâm $H$ và $P$ là một điểm di động trên $AH$. Đường thẳng vuông góc với $AC$ tại $C$ cắt $BP$ tại $M$, đường thẳng vuông góc với $AB$ tại $B$ cắt $CP$ tại $N.$ Gọi $K$ là hình chiếu của $A$ trên $MN$. Chứng minh $\angle BKC+\angle MAN$ không đổi.

Bài 5.
Tìm tất cả đa thức $P(x),Q(x)$ có hệ số nguyên và thỏa mãn điều kiện:
Với dãy số $({{x}_{n}})$ xác định bởi: ${{x}_{0}}=2014,{{x}_{2n+1}}=P({{x}_{2n}}),{{x}_{2 n}}=Q({{x}_{2n-1}})$ với $n\ge 1$ thì mỗi số nguyên dương $m$ là ước của một số hạng khác 0 nào đó của dãy $({{x}_{n}})$.

Bài 6.
Cho $m,n,p$ là các số tự nhiên không đồng thời bằng 0. Không gian tọa độ được chia thành các mặt phẳng song song cách đều nhau. Một cách điền vào mỗi khối lập phương đơn vị một trong các số từ 1 đến 60 được gọi là cách điền Điện Biên nếu thỏa mãn: trong mỗi hình hộp chữ nhật với các mặt trên các hệ mặt đã cho và tập hợp độ dài ba cạnh xuất phát từ một đỉnh là $\left\{ 2m+1,2n+1,2p+1 \right\}$. Khối lập phương đơn vị có tâm trùng với tâm của hình hộp chữ nhật được điền số bằng trung bình cộng của các số điền ở tâm của 8 hình lập phương ở các góc của hình hộp đó. Hỏi có tất cả bao nhiêu cách điền Điện Biên?
Những cách điền là giống nhau nếu các số được điền vào các khối lập phương đơn vị có cùng tọa độ trong các cách này đều giống nhau.

Cập nhật đề bài 6, nguồn: blackholes.
[RIGHT][I][B]Nguồn: MathScope.ORG[/B][/I][/RIGHT]
 
File Kèm Theo
Kiểu File : pdf De Thi TST Ngay 2.pdf (165.2 KB, 248 lần tải)
__________________
Sự im lặng của bầy mèo

thay đổi nội dung bởi: huynhcongbang, 26-03-2014 lúc 05:23 PM
huynhcongbang is offline   Trả Lời Với Trích Dẫn
The Following 8 Users Say Thank You to huynhcongbang For This Useful Post:
BlackSelena (26-03-2014), kimlinh (27-03-2014), lehuu (26-03-2014), lupanh7 (26-03-2014), magician_14312 (26-03-2014), mathandyou (26-03-2014), thaygiaocht (26-03-2014), thiendieu96 (26-03-2014)
Old 26-03-2014, 04:54 PM   #14
kien10a1
+Thành Viên+
 
kien10a1's Avatar
 
Tham gia ngày: Feb 2011
Đến từ: Vĩnh Yên- Vĩnh Phúc
Bài gởi: 371
Thanks: 43
Thanked 263 Times in 153 Posts
Gửi tin nhắn qua Yahoo chát tới kien10a1
Trích:
Nguyên văn bởi huynhcongbang View Post
Đề thi ngày 2.


Bài 5.
Tìm tất cả đa thức $P(x),Q(x)$ có hệ số nguyên và thỏa mãn điều kiện:
Với dãy số $({{x}_{n}})$ xác định bởi: ${{x}_{0}}=2014,{{x}_{2n+1}}=P({{x}_{2n}}),{{x}_{2 n}}=Q({{x}_{2n-1}})$ với $n\ge 1$ thì mỗi số nguyên dương $m$ là ước của một số hạng khác 0 nào đó của dãy $({{x}_{n}})$.
Nêu tí cảm nhận về bài này.
Nhận xét: với đa thức $R(x) $ hệ số nguyên, nếu bậc R lớn hơn 1, thì với x có trị tuyệt đối đủ lớn, ta có $\left |R(x) \right |>2\left | x \right | $

Nếu có một trong hai đa thức $P, Q $ là hằng, dễ có điều vô lý.
Giả sử, trong hai đa thức P(x) và Q(x) có một đa thức có bậc lớn hơn 1, có thể coi là $Q(x) $, khi đó bậc của $Q(P(x)) $ cũng lớn hơn 1.

Suy ra, với mọi x lớn hơn N thì $\left |Q(P(x)) \right |>\left | P(x) \right |+\left | x \right | $
Bây giờ, xét dãy số thỏa mãn. Vì trong dãy có số chia hết cho m lớn bất kỳ khác 0, nên tồn tại $\left | x_{i} \right | $ lớn tùy ý, không khó từ đó suy ra có j mà $\left | x_{2j} \right |>N+1 $ và $x_{2j} $ là số có giá trị tuyệt đối lớn nhất trong 2j số đầu tiên.

Từ nhận xét trên suy ra chọn $m=\left |x_{2j+2}-x{2j} \right | $ thì trong 2j+2 số đầu của dãy, không số nào chia hết cho m. Và ta lại có $x_{2k+2}-x_{2k}\vdots x_{2j+2}-x_{2j}
x_{2k+3}-x_{2k+1}\vdots x_{2j+2}-x_{2j} $
với mọi k>j, nên dãy không có số nào chia hết cho m.
Như vậy, P, Q đều có bậc 1.
Lập luận tương tự, ta thấy $P,Q $ không thể cùng có hệ số cao nhất có trị tuyệt đối lớn hơn 1.
Xét P có hệ số cao nhất là 2, bằng cách như trên, ta suy ra nếu P(x) có dạng $2x-a $ thì $Q(x) $ phải có dạng $x+a $, thay vào không khó để thấy vô lí.
Cuối cùng là $P,Q $ đều có hệ số cao nhất là $1 $ hoặc $-1 $.
Nếu hai hệ số cao nhất này đối nhau, dễ thấy dãy tuần hoàn.
Ta chỉ còn xét $P(x)=x+b,Q(x)=x+d $ hoặc $P(x)=-x+b $ và $Q(x)=-x+d $
Đến đây thì không khó để làm tiếp rồi.

[RIGHT][I][B]Nguồn: MathScope.ORG[/B][/I][/RIGHT]
 
__________________
Quay về với nơi bắt đầu
kien10a1 is offline   Trả Lời Với Trích Dẫn
The Following 8 Users Say Thank You to kien10a1 For This Useful Post:
bachhammer (29-03-2014), dangvip123tb (26-03-2014), huynhcongbang (27-03-2014), Kém Toán (26-03-2014), kimlinh (27-03-2014), let (27-03-2014), nam8298 (27-03-2014), thiendieu96 (26-03-2014)
Old 26-03-2014, 05:50 PM   #15
Nguyen Van Linh
Moderator
 
Tham gia ngày: Aug 2009
Đến từ: Hà Nội
Bài gởi: 277
Thanks: 69
Thanked 323 Times in 145 Posts
Trích:
Nguyên văn bởi huynhcongbang View Post
Đề thi ngày 2.

Bài 4.
a. Cho tam giác $ABC$ có đường cao $AD$ và $P$ là một điểm di động trên $AD$. Các đường thẳng $PB$ và $AC$ cắt nhau ở $E$, các đường thẳng $PC$ và $AB$ cắt nhau ở $F.$ Giả sử tứ giác $AEDF$ nội tiếp. Chứng minh rằng $$\frac{PA}{PD}=(\tan B+\tan C)\cot \frac{A}{2}.$$ b. Cho tam giác $ABC$ có trực tâm $H$ và $P$ là một điểm di động trên $AH$. Đường thẳng vuông góc với $AC$ tại $C$ cắt $BP$ tại $M$, đường thẳng vuông góc với $AB$ tại $B$ cắt $CP$ tại $N.$ Gọi $K$ là hình chiếu của $A$ trên $MN$. Chứng minh $\angle BKC+\angle MAN$ không đổi.
a) Gọi $L$ là giao của $EF$ và $BC$ suy ra $(LDBC)=-1$, từ đó $DA$ là phân giác $\angle FDE$. Tứ giác $AFDE$ nội tiếp nên $\angle AFE=\angle ADE=\angle ADF=\angle AEF$. Do đó nếu gọi $H$ là giao điểm thứ hai của $(AEF)$ với $BC$ thì $AH$ là phân giác $\angle BAC.$
Ta có $(\tan B+\tan C)\cot \frac{A}{2}=(\dfrac{HF}{FB}+\dfrac{HE}{EC})\cdot \dfrac{AF}{FH}=\dfrac{AF}{FB}+\dfrac{AE}{EC}.$
Mặt khác, áp dụng định lý Menelaus suy ra $\dfrac{AP}{PD}\cdot\dfrac{BD}{BC}\cdot\dfrac{EC}{ EA}=1$
Từ đó $\dfrac{AP}{PD}=\dfrac{AE}{EC}\cdot\dfrac{BC}{BD}. $
Ta cần chứng minh $\dfrac{AE}{EC}\cdot\dfrac{BC}{BD}=\dfrac{AF}{FB}+ \dfrac{AE}{EC}$ hay $\dfrac{AE}{EC}\cdot\dfrac{DC}{BD}=\dfrac{AF}{FB}$ , điều này hiển nhiên đúng theo định lý Céva cho tam giác $ABC$ và 3 điểm $D,E,F.$

b) Gọi $I$ là giao của $NB$ và $MC$ thì $I$ nằm trên $(O)$ (điểm đối xứng với $A$ qua $O$). Chú ý rằng $AKBN$ và $AKCM$ nội tiếp nên $\angle NAM+\angle BKC=\angle KBI+\angle KCI+\angle BKC=360^\circ-\angle BIC=const.$

Rõ ràng câu b không cần đến $P$ nằm trên $AH$. Theo nhận xét của mình thì đề cho 2 câu a,b như thế này là không hay, thậm chí 2 câu không liên quan gì đến nhau. Để ý bài hình của VN bao giờ cũng phải có câu a,b. Sao cứ phải làm thế nhỉ?
[RIGHT][I][B]Nguồn: MathScope.ORG[/B][/I][/RIGHT]
 
File Kèm Theo
Kiểu File : pdf TST4.2014.pdf (15.3 KB, 153 lần tải)

thay đổi nội dung bởi: Nguyen Van Linh, 26-03-2014 lúc 05:54 PM
Nguyen Van Linh is offline   Trả Lời Với Trích Dẫn
The Following 6 Users Say Thank You to Nguyen Van Linh For This Useful Post:
bachhammer (29-03-2014), dangvip123tb (26-03-2014), dung_toan78 (26-03-2014), huynhcongbang (27-03-2014), kimlinh (27-03-2014), thiendieu96 (26-03-2014)
Trả lời Gởi Ðề Tài Mới

Bookmarks

Ðiều Chỉnh
Xếp Bài

Quuyền Hạn Của Bạn
You may not post new threads
You may not post replies
You may not post attachments
You may not edit your posts

BB code is Mở
Smilies đang Mở
[IMG] đang Mở
HTML đang Tắt

Chuyển đến


Múi giờ GMT. Hiện tại là 11:35 AM.


Powered by: vBulletin Copyright ©2000-2024, Jelsoft Enterprises Ltd.
Inactive Reminders By mathscope.org
[page compression: 140.82 k/157.57 k (10.63%)]